Convergence of a recursive sequence

Click For Summary
The discussion centers on the convergence of the recursive sequence defined by a given initial value a_1 in the natural numbers. It is established that if the sequence converges to L, then L must equal 1, which is only the case when a_1 equals 1. For a_1 greater than or equal to 2, the sequence is shown to be bounded below by 2, indicating it cannot converge to 1. The participants emphasize the necessity of demonstrating that a_{n+1} is greater than a_n for all n, reinforcing the argument against convergence for initial values greater than 1. Ultimately, the conclusion is that the sequence does not necessarily converge when a_1 is in the natural numbers.
Mr Davis 97
Messages
1,461
Reaction score
44

Homework Statement


With ##a_1\in\mathbb{N}## given, define ##\displaystyle {\{a_n\}_{n=1}^\infty}\subset\mathbb{R}## by ##\displaystyle {a_{n+1}:=\frac{1+a_n^2}{2}}##, for all ##n\in\mathbb{N}##.

Homework Equations

The Attempt at a Solution


We claim that with ##a_1 \in \mathbb{N}##, the sequence does not necessarily converge. First, we note that if the sequence did converge to ##L##, then ##L = \frac{L^2+1}{2} \implies (L-1)^2=0 \implies L=1## (we used the fact that since ##(a_{n+1})## is a subsequence of ##(a_n)##, ##\lim a_n = \lim a_{n+1} = L##). We now split the proof up into two cases:Case 1: ##a_1=1##. In this case, for all positive integers ##n##, ##a_n = 1##. Hence ##\lim a_n = 1##.

Case 2: ##a_1 \ge 2##. First, we show that ##(a_n)## is bounded below by 2. That is, we want to show that ##\forall n \in \mathbb{N}##, ##a_n \ge 2##. We proceed by induction. The base case clearly holds. Suppose that for some ##k \in \mathbb{N}## we have ##a_k \ge 2##. Then ##a_{k+1} = \frac{a_k^2+1}{2} \ge \frac{4+1}{2} = \frac{5}{2} \ge 2##. So the sequence is bounded below by ##2##. Hence, the sequence does not converge, for if it did, the terms of the sequence would get arbitrarily close to ##1## for large ##n##.
 
Physics news on Phys.org
The sequence ##(3,3,3,3,3, \ldots)## is bounded from below by ##2##, yet it converges. You should show ##a_{n+1}>a_n## for all ##n## and ##a_1>1\,.##
 
fresh_42 said:
The sequence ##(3,3,3,3,3, \ldots)## is bounded from below by ##2##, yet it converges. You should show ##a_{n+1}>a_n## for all ##n## and ##a_1>1\,.##
But the sequence ##(3,3,3,3, \dots)## does not converge to ##1##. What I thought I showed was that if the sequence did converge, it would have to converge to ##1##, and that in the case that ##a_1 \ge 2##, the sequence is bounded below by 2, and so couldn't possibly converge to ##1##. Hence it doesn't necessarily converge with ##a_1 \in \mathbb{N}##.
 
Mr Davis 97 said:
But the sequence ##(3,3,3,3, \dots)## does not converge to ##1##. What I thought I showed was that if the sequence did converge, it would have to converge to ##1##, and that in the case that ##a_1 \ge 2##, the sequence is bounded below by 2, and so couldn't possibly converge to ##1##. Hence it doesn't necessarily converge with ##a_1 \in \mathbb{N}##.
You're right. I forgot this. Btw., the same argument as for ##L=1## also shows ##a_{n+1}>a_n\,.##
 
Question: A clock's minute hand has length 4 and its hour hand has length 3. What is the distance between the tips at the moment when it is increasing most rapidly?(Putnam Exam Question) Answer: Making assumption that both the hands moves at constant angular velocities, the answer is ## \sqrt{7} .## But don't you think this assumption is somewhat doubtful and wrong?

Similar threads

  • · Replies 2 ·
Replies
2
Views
2K
  • · Replies 8 ·
Replies
8
Views
2K
  • · Replies 1 ·
Replies
1
Views
2K
  • · Replies 5 ·
Replies
5
Views
3K
Replies
3
Views
2K
  • · Replies 7 ·
Replies
7
Views
2K
Replies
4
Views
2K
  • · Replies 2 ·
Replies
2
Views
2K
  • · Replies 2 ·
Replies
2
Views
2K
  • · Replies 3 ·
Replies
3
Views
1K